Search found 258 matches


The ans is A When 4 is divided by 3 remainder is 1 when 7n is divided by 3 remainder = 2 (this u can find by plugging values of n where n + 1 is divisible by 3. Also since it is given that n is a positive integer and n+1 is divisible by 3 u can rule out the possibility of n being 0) So remainder whe...

by ri2007

Fri Nov 30, 2007 12:04 pm
Forum: Data Sufficiency
Topic: GMAT PREP DIVISIBILTY ?
Replies: 1
Views: 1358

This one has been discussed in the forum many times before. But in case u have ur exam soon here is the short ans With statement 1 & 2 combined u have the following info angle rqs = angle rsq = z so angle qrs = 180 - 2z the same with the other triangle angle tsu = angle tus = y so angle rtu = 18...

by ri2007

Fri Nov 30, 2007 11:53 am
Forum: Data Sufficiency
Topic: GMAT PREP DS?
Replies: 1
Views: 1251

I agree with Suyog on the approach but my ans is E

Amt of Petrol use = 25/3 = 8.33
so ans = (25/3) / 12 = 25/36

pls confirm

by ri2007

Fri Nov 30, 2007 11:25 am
Forum: Problem Solving
Topic: GMAT PS QUES?
Replies: 3
Views: 1685

I did this by plugging the values So , if u solve the equation in the question stem u get 10 ( x+2y) / (x+y) = k Now just plug the value of each ans choice instead of K you will see that the only solution where x<y and both are positive is with 18 as the ans. Is there a better way to do this?

by ri2007

Fri Nov 30, 2007 10:58 am
Forum: Problem Solving
Topic: PS
Replies: 1
Views: 1405

I would go with 1 & 3 (last choice) If n = 1, K = 1, so no of factors is 1, which is odd if n = 2, K = 4, so no of factors = 3, which is both odd and a prime number. However if K is a square of n, it cannot have even numbers. Here is my thinking. say k = x^2y^2 no of factors = (2+1) (2+1) both a...

by ri2007

Thu Nov 29, 2007 7:42 am
Forum: Problem Solving
Topic: Math PS
Replies: 5
Views: 1730

Assume total amount of hospital expenses last year = x
so total amount paid = 0.8x
which is also = 408 + 0.2x

so solve for 408 + 0.2x = 0.8x

by ri2007

Thu Nov 29, 2007 6:29 am
Forum: Problem Solving
Topic: Math, PS
Replies: 1
Views: 1647

i agree with the apporach camitava has taken but my ans is 3/2

pls confirm

by ri2007

Thu Nov 29, 2007 6:19 am
Forum: Problem Solving
Topic: Gmat Prep Ratio?
Replies: 4
Views: 1729

How is B sufficient

If u have 3 men in the group u will have 7 women.

So P(both women) = 7/10 * 6/9 = 7/15 which is less than half.

Pls explain

by ri2007

Thu Nov 29, 2007 6:07 am
Forum: Data Sufficiency
Topic: probability DS
Replies: 3
Views: 1489

The ans should be C

The lowest values for which a = 2b and a + b > 0 are

b = 0.4 and a = 0.8.

if u put these values or any values greter than then in all cases u will find ab/a+b is less than ab

pls confirm

by ri2007

Wed Nov 28, 2007 2:08 pm
Forum: Data Sufficiency
Topic: DS 2
Replies: 11
Views: 2263

ans should be C

both X&Y have the same sign, and X&Z have the same sign. If X is negative even Y and Z are negative. If all 3 are negative then XYZ<0>0

so all three have to be positive

by ri2007

Wed Nov 28, 2007 12:50 pm
Forum: Data Sufficiency
Topic: GMAT PREP ?
Replies: 3
Views: 1725

my ans was C

SP = P + Purchase Price (PP)

Combined we have

SP = X
Profit = 50 = 0.2X

So SP = 250

pls confirm

by ri2007

Wed Nov 28, 2007 12:46 pm
Forum: Data Sufficiency
Topic: DS question
Replies: 4
Views: 4235

total ways in which u can select 1 from 1000 = 1000 total ways in which u can select 1 from 800 = 800 total ways u can select 1 junior and 1 senior = 1000 * 800 total ways u can select 1 sibiling from one class = 60 total ways u can select the same siblings pair from the other class = 1 so ans = 60*...

by ri2007

Wed Nov 28, 2007 12:42 pm
Forum: Data Sufficiency
Topic: GMAT QUES Prob ?
Replies: 3
Views: 1223

ans should be A

Mean = Sum / No of terms.

Since the sum is the same for both the sets and the avg of one is less than the other. The one with the lower avg will have greater terms.

pls confirm

by ri2007

Wed Nov 28, 2007 12:38 pm
Forum: Data Sufficiency
Topic: Gmat Prep Ques
Replies: 3
Views: 1263

Samir

As always an amazing explaination. Thanks a lot

by ri2007

Tue Nov 27, 2007 6:55 am
Forum: Data Sufficiency
Topic: help~
Replies: 5
Views: 1920

Thanks so much for ur immediate reply. Another question if u dont mind.

I got how u narrowed it down to n = any prime number greater than 3

but how did u figure out that the sq of a prime number - 1 is a multiple of 24 and so r = 0?

is there a rule or some thing,

thanks again

by ri2007

Mon Nov 26, 2007 8:28 pm
Forum: Data Sufficiency
Topic: help~
Replies: 5
Views: 1920